Cute diophantine
by TestX01, Apr 24, 2025, 2:36 AM
Find all sequences of four consecutive integers such that twice their product is perfect square minus nine.
Complicated FE
by XAN4, Apr 23, 2025, 11:53 AM
Find all solutions for the functional equation
, in which
: 
Note: the solution is actually quite obvious -
, but the proof is important.
Note 2: it is likely that the result can be generalized into a more advanced questions, potentially involving more bash.



Note: the solution is actually quite obvious -

Note 2: it is likely that the result can be generalized into a more advanced questions, potentially involving more bash.
Stronger inequality than an old result
by KhuongTrang, Aug 1, 2024, 3:13 PM
Problem. Find the best constant
satisfying
holds for all 

![$$(ab+bc+ca)\left[\frac{1}{(a+b)^{2}}+\frac{1}{(b+c)^{2}}+\frac{1}{(c+a)^{2}}\right]\ge \frac{9}{4}+k\cdot\frac{a(a-b)(a-c)+b(b-a)(b-c)+c(c-a)(c-b)}{(a+b+c)^{3}}$$](http://latex.artofproblemsolving.com/f/6/e/f6ed10f7fff1cc94edd8f451e75718a0916a8bfa.png)

This post has been edited 2 times. Last edited by KhuongTrang, Aug 2, 2024, 6:43 AM
Something nice
by KhuongTrang, Nov 1, 2023, 12:56 PM
Problem. Given
be non-negative real numbers such that
Prove that




This post has been edited 2 times. Last edited by KhuongTrang, Nov 19, 2023, 11:59 PM
\frac{1}{9}+\frac{1}{\sqrt{3}}\geq a^2+\sqrt{a+ b^2} \geq \frac{1}{4}
by sqing, Oct 3, 2023, 8:11 AM
Let
and
Prove that








This post has been edited 2 times. Last edited by sqing, Oct 3, 2023, 8:55 AM
Easy IMO 2023 NT
by 799786, Jul 8, 2023, 4:53 AM
Determine all composite integers
that satisfy the following property: if
,
,
,
are all the positive divisors of
with
, then
divides
for every
.










This post has been edited 7 times. Last edited by v_Enhance, Sep 18, 2023, 12:33 AM
Reason: minor latex pet peeve
Reason: minor latex pet peeve
hard binomial sum
by PRMOisTheHardestExam, Mar 6, 2023, 5:35 PM
x_1x_2...x_(n+1)-1 is divisible by an odd prime
by ABCDE, Jul 7, 2016, 9:45 PM
Let
and
be positive integers such that
. Define
for
. Prove that if all the numbers
are integers, then
is divisible by an odd prime.







This post has been edited 1 time. Last edited by ABCDE, Jul 7, 2016, 9:47 PM
IMO 2012/5 Mockup
by v_Enhance, Jul 30, 2013, 5:19 AM
Let
be a scalene triangle with
, and let
be the foot of the altitude from
. Let
be a point in the interior of the segment
. Let
be the point on the segment
such that
. Similarly, let
be the point on the segment
such that
. The circumcircle of triangle
intersects segment
at a second point
(other than
). Prove that
.

















This post has been edited 1 time. Last edited by v_Enhance, May 7, 2015, 1:33 AM
Reason: 90\dg should be 90^{\circ}
Reason: 90\dg should be 90^{\circ}
♪ i just hope you understand / sometimes the clothes do not make the man ♫ // https://beta.vero.site/
Archives

















































































Shouts
Submit
91 shouts
Contributors
Tags
About Owner
- Posts: 583
- Joined: Dec 16, 2006
Blog Stats
- Blog created: May 17, 2010
- Total entries: 327
- Total visits: 356648
- Total comments: 368
Search Blog